Triangles PQR and PST are similar to each other. Find the scale factor from the figure.


2
3
4
1.5

Answers

Answer 1

2+3=5 and 4+1.5=2SO BY ADDIDING IS 9


Related Questions

Find the value of x in each triangle. Please help me <33

Answers

Answer:

x = 11

Step-by-step explanation:

The sum of the 3 angles in a triangle = 180°

sum the 3 angles and equate to 180

5x + 4 + 6x - 3 + 4x + 14 = 180 ← collect like terms on left side

15x + 15 = 180 ( subtract 15 from both sides )

15x = 165 ( divide both sides by 15 )

x = 11

The coordinates of the turning point of the parabola whose equation is y = x2 — 6x + 8 are A. (3, 35) C. (-3, 35) B. (-3, —1) D. (3. —1)

Answers

Answer:

D. (3. —1)

Step-by-step explanation:

Vertex of a quadratic function:

Suppose we have a quadratic function in the following format:

[tex]f(x) = ax^{2} + bx + c[/tex]

It's vertex is the point [tex](x_{v}, y_{v})[/tex]

In which

[tex]x_{v} = -\frac{b}{2a}[/tex]

[tex]y_{v} = -\frac{\Delta}{4a}[/tex]

Where

[tex]\Delta = b^2-4ac[/tex]

If a<0, the vertex is a maximum point, that is, the maximum value happens at [tex]x_{v}[/tex], and it's value is [tex]y_{v}[/tex].

Turning point of a quadratic function:

The turning point of a quadratic function is the vertex.

y = x2 — 6x + 8

This means that [tex]a = 1, b = -6, c = 8[/tex]

Now, we find the vertex.

[tex]x_{v} = -\frac{b}{2a} = -\frac{-6}{2(1)} = 3[/tex]

[tex]\Delta = b^2-4ac = (-6)^2-4(1)(8) = 36 - 32 = 4[/tex]

[tex]y_{v} = -\frac{4}{4(-1)} = -1[/tex]

(x,y) = (3,-1), and the correct answer is given by option D.

If K(5,-2) is rotated 90°, what will be the coordinates of its image?
K' (2,5)
K'(-2,-5)
K'(-5, 2)
K'(2,-5)

Answers

K’(-5,2) is the answer

What is the reflection of point P (2, -7) over the x-axis

Answers

Answer:

(2,7) I think

Step-by-step explanation:

Answer:

(2,7)

Step-by-step explanation:

It's funny, this is the exact topic I just learned.

Ok, 2,-7 would be in the fourth quadrant (IV).

So using this information if we want to reflect it over the x axis, the point would just got over the x axis in quadrant 1. We know that quadrant 1 is (++) so the ordered pair MUST be positive. Hence the answer of (2,7)

Hope this helps!

Which equation represents a line which is perpendicular to the line 6x+y=-4

Answers

Answer:

y = 1/6x - 4

Step-by-step explanation:

y = -6x - 4  

current slope is -6 so the slope of a line perpendicular would be 1/6

y = 1/6x - 4

For the arithmetic sequence beginning with the terms {-2, 0, 2, 4, 6, 8...}, what is the sum of the first 18 terms?

Answers

Answer:

The sum of the first 18 terms is 270.

Step-by-step explanation:

First the 18th term is calculated which is gotten using the following steps below:

an = a1 + d ( n - 1 )

Where, an =18th term

a1= first term (-2)

d = difference

n = 18

Therefore, 18th term

an= -2 + 2 (18 - 1)

= -2 + 2 (17)

= -2 + 34

18th term = 32

To calculate the sum of the first 18th terms the following steps are used,

Sn = n(a1 + an)/2

Where,

Sn = sum of 18th term

n = 18

a1 = -2

an = 18 term

Therefore,

Sn = 18 (- 2+ 32)/2

= 18 (30)/2

= 18 ×30 /2

= 540/2

= 270

a bookcase is 500mm wide how many of these bookcase will fill a
space 2m wide​

Answers

I think 4 but I could be wrong

Angle B = ?
Need help.

Answers

Answer:

angle B = 33.42

Step-by-step explanation:

take angle B as reference angle

using tan rule

tan B = opposite/adjacent

tan B = 6/9

tan B = 0.66

B = [tex]tan^{-1} (0.66)[/tex]

B = 33.424

B = 33.42

A group of 31 friends gets together to play a sport. First, people must be divided into teams. Each team has to have exactly 3 players, and no one can be on more than one team. How many teams can they make?

Answers

10 because 30 divided by 3 is 10

Answer:

10, 1 remaining

Step-by-step explanation:

Each team can have 10 people but there will be a remaining person that will not be able to participate.

31 ÷ 3= 10.33333333

Hope this helped :)

Mixing 4 ml of red paint and 15 ml of yellow paint makes orange paint. How much red would be needed if you use 1500 ml of yellow paint?

Answers

Answer:

Amount of red paint needed for 1500 ml yellow paint = 400 ml

Step-by-step explanation:

Given:

ration of red paint to yellow paint = 4 ml to 15 ml

Find:

Amount of red paint needed for 1500 ml yellow paint

Computation:

Amount of red paint needed for 1500 ml yellow paint = [ration of red paint to yellow paint]1500

Amount of red paint needed for 1500 ml yellow paint = [4/15]1500

Amount of red paint needed for 1500 ml yellow paint = [4]100

Amount of red paint needed for 1500 ml yellow paint = 400 ml

WILL GIVE BRANLIEST!!! pls help tysm!!!

Answers

Try this I could have misunderstood the question but this it’s what I got don’t mind the hand writing

Honestly, I know these answers I just wanna see if the answers I got are correct.

Answers

Answer:

4. 78,020,008 4. Seventy-eight million, twenty thousand, eight

5. 20,011,012 5. 20,000,000 + 11,000 + 12

6. 50,000 7. 900 8. 70,000,000 9. 30,000,000

A circle has a center at (-2, 5) and a radius of bold 3 square root of bold 2units. What is the equation of the circle in standard form?

Answers

Given:

The center of the circle = (-2,5)

The radius of the circle = [tex]3\sqrt{2}[/tex] units.

To find:

The equation of the circle is standard form.

Solution:

The standard form of a circle is:

[tex](x-h)^2+(y-k)^2=r^2[/tex]               ...(i)

Where, (h,k) is the center of the circle and r is the radius of the circle.

It is given that the center of the circle is (-2,5). So, [tex]h=-2,\ k=5[/tex].

The radius of the circle is [tex]3\sqrt{2}[/tex] units. So, [tex]r=3\sqrt{2}[/tex].

Putting [tex]h=-2,\ k=5[/tex] and [tex]r=3\sqrt{2}[/tex] in (i), we get

[tex](x-(-2))^2+(y-(5))^2=(3\sqrt{2})^2[/tex]

[tex](x+2)^2+(y-5)^2=18[/tex]

Therefore, the equation of the circle is [tex](x+2)^2+(y-5)^2=18[/tex].

which pairs of angles in the figure below are vertical angles select all that apply

Answers

Answer:the left one

Step-by-step explanation:

Can I please get some help? I would really appreciate it.

Answers

It’ should be 2nd I’m sure

Gary buys a skateboard for
$145. What's the total cost
including 6% sales tax?

Answers

Answer:

$8.6

Step-by-step explanation:

The total tax is 8 dollars and 60 cents

Answer:

$153.70

Step-by-step explanation:

Sales tax is applied to things that are sold by a retailer. So if it says 6% tax is levied on the item, that means the total cost will be the initial cost of skateboard ($145) + sales tax (6% of 145)  = 145 + 8.7 = $153.70

i’m horrible at math, can anyone help?

Answers

Answer: no I'm actually horrible at math too sorry

y = 3x - 1
3x - y = 9
X=
A) x = 123, y = 4 4
B) x=3, y = 0
C) No solution
D) Infinitely many solutions

Answers

Answer:

no solution.

Step-by-step explanation:

given that, y = 3x-1

so,

in equation, 3x-y = 9

putting the value of y here,

3x-(3x-1) = 9

3x-3x+1 = 9

3x and 3x is cancelled.

hence, no solution

A horizontal line passes through the point (5, –1). Which point is also on this line?
(0, 0)
(–1, 5)
(5, –4)
(–2, –1)

Answers

Answer:

Step-by-step explanation:

A horizontal line is a "y =  " line. So the horizontal line that goes through the point given has the equation y = -1 (-1 being the y coordinate of the point). That means that another point on this line, y = -1, will also have to have a y coordinate of -1. The only one is the last one, (-2, -1).

Answer:

D

Step-by-step explanation:

I just took the test

12) Compare. Use <, >, or =. 665.5 - 281.7_1
373.8

Answers

Answer:

383.8   =  383.8

Step-by-step explanation:

665.5-281.7   ?   373.8

383.8   =  383.8

A teacher is buying supplies for two art classes. For class 1, the teacher buys 24 tubes of paint, 12 brushes, and 17 canvasses. For class 2, the teacher buys 20 tubes of paint, 14 brushes, and 15 canvasses. Each tube of paint costs $3.35, each brush costs $1.75, and each canvas costs $4.50. Compute the following matrix multiplication to determine how much the teacher spent on each class.

Answers

Class 1 =$177.90
Class 2 =$159


Class 1

24 tube of paint / $3.35 each
12 brushes/ $1.75 each
17 canvases/$4.50 each
Total cost of paint= $80.40
Total cost of brushes=$21
Total cost of canvases= $76.50
Total spent on class 1 = $177.90

Class 2

20 tubes of paint/$3.35 each
14 brushes/$1.75 each
15 canvases/ $4.50 each
Total cost of paint=$67
Total cost of brushes =$24.50
Total cost of canvases=$67.50
Total spent on class 2 = $159

PLSSS HELP!! I need the answer m.

Answers

6578 i don’t know don’t ask me bit

A student skipped a step when she tried to convert 15 hours to seconds, and
she got the following incorrect result:
15 hours
60 seconds
1 minute
= 900 seconds
What conversion ratio did she skip in this multiple-step conversion?
O A.
60 seconds
1 minute
B.
60 minutes
1 hour
C.
1 hour
60 minutes
D.
1 minute
60 seconds

Answers

Answer:

B.

60 minutes

1 hour

Una maestra esta comprando lapices para su salon de clases. Si una docena de lapices cuesta $0.79 puede comprar lapices con $5?

Answers

Answer:

Cantidad total de lapices= 76

Step-by-step explanation:

Dada la siguiente información:

Costo por docena= $0.79

Presupuesto total= $5

Para calcular la cantidad total de lapiceras que se pueden comprar con $5, debemos usar la siguiente formula:

Cantidad total de lapices= presupuesto total / costo por docena

Cantidad total de lapices=  5 / 0.79

Cantidad total de lapices= 6.33

Para ser mas exactos:

0.33*12= 4

Cantidad total de lapices= 6*12 + 4

Cantidad total de lapices= 76


How does the range of g(x)=6/x compare with the range of the parent function f(x)=1/x?

Answers

Answer:

The range of both [tex]f(x)[/tex] and [tex]g(x)[/tex] is all nonzero real numbers.

Step-by-step explanation:

The range of the function is the set of real number for [tex]y[/tex]. The rational function of the form [tex]y = \frac{a}{x}[/tex], where [tex]a[/tex] is a constant, has a range represented by the subset of nonzero real numbers. This is: [tex]Ran\{f(x)\} = \mathbb{R}-\{0\}[/tex]

Hence, the correct answer is: The range of both [tex]f(x)[/tex] and [tex]g(x)[/tex] is all nonzero real numbers.

Write the point-slope form of an equation of the line through the points (6, -1) and (5, -7).

Answers

Answer:

y + 1 = 6(x - 6)

Step-by-step explanation:

y2 - y1 / x2 - x1

-7 - (-1) / 5 - 6

-6/-1

= 6

y + 1 = 6(x - 6)

% of $4 = $1
What is

Answers

Answer:

25 percent is the answer to your question.

Find the total area

Answers

Answer:

I think it's 33 I don't know

A motorcycle cost $12,000 when it was purchased. The value of a motorcycle decreases by 6% each
year. Find the rate of decay each month and select the correct answer below.

-0.005143%
-0.5143%
-0.005%
-0.5%

Answers

Answer:

the correct answer is -0.5%


A line has a slope of -1/4 and passes through point
(-5/4, 1)
What is the equation of the line?

Answers

Answer:

y = -1/4 x+11/16

Step-by-step explanation:

The slope intercept form of a line is

y = mx+b where m is the slope and b is the y intercept

y = -1/4x+b

Substitute the point in for x and y

1 = -1/4(-5/4) +b

1 = 5/16 +b

16/16 = 5/16 +b

Subtract 5/16 from each side

11/16 = b

y = -1/4 x+11/16

Answer:

y = - [tex]\frac{1}{4}[/tex] x + [tex]\frac{11}{16}[/tex]

Step-by-step explanation:

The equation of a line in slope- intercept form is

y = mx + c ( m is the slope and c the y- intercept )

Here m = - [tex]\frac{1}{4}[/tex] , then

y = - [tex]\frac{1}{4}[/tex] x + c ← is the partial equation

To find c substitute ( - [tex]\frac{5}{4}[/tex], 1 ) into the partial equation

1 = [tex]\frac{5}{16}[/tex] + c ⇒ c = 1 - [tex]\frac{5}{16}[/tex] = [tex]\frac{11}{16}[/tex]

y = - [tex]\frac{1}{4}[/tex] x + [tex]\frac{11}{16}[/tex]

Other Questions
Alex is planning to surround his pool ABCD with a single line of tiles. How many units of tile will he need to surround his pool? Round your answer to the nearest hundredth.Answers: 8.9610.4813.4220.42 Select all the correct imagesSelect the atoms that belong to the same element With regards to the three levels of prevention/ intervention: _____________________ prevention minimizes the severity of the disease or injury once it has occurred. This can include health screenings such as mammograms, and colonoscopies for those in the appropriate age/risk category. Which of these events were effects of the Homestead Act? Check all of the boxes that apply. Example 3:In how many ways can a supermarket manager display 5 brands of cerealsin 3 spaces on a shelf?Solution: BEING TIMED! 10 pointsIn what ways can Washington voters participate in direct democracy? Check all that apply. MULTI CHOICEby being public servantsby creating initiativesby participating in protestsby running for public officeby voting in recall electionsby voting in referendums Which of these building materials is a renewable resource?A. Stainless steelB. BambooC. GraniteD. Limestone A colony contains 1500 bacteria. The population increases at a rate of 115% each hour. If x represents the number of hours elapsed, which function represents the scenario?f(x) = 1500(1.15)x f(x) = 1500(115)xf(x) = 1500(2.15)xf(x) = 1500(215)x Jalen says that the height, radius, and diameter of a cone lie entirely on the base of the cone. What is Jalens error?The height does not lie on the base because it is perpendicular to the base.The radius does not lie on the base because it contains the center of the base.The diameter does not lie on the base because it contains a radius of the circle.The base does not contain the height, the radius, or the diameter because it is a circle. What is the value of x? Which term describes habit-forming drugs that are regulated by the Drug Enforcement Agency?a. beta blockersb. regulated substancesc. macrolidesd. controlled substances Select the sentence below that uses the modifier correctly AND is punctuated correctly.1) Today, I saw a man riding a classic black motorcycle with a broken leg. 2) I slipped on the banana peel running through the parking lot.3) While cleaning in the attic, Brutus found a woman's silver necklace. 4) After eating sixteen hot dogs in the competition, nausea swept over Jim Bob A rectangular field is covered by circular sprinklers asshown in the diagram. What percentage of the field is notbeing watered by the sprinklers? Three people build a wall in 10 days. How long would it take five people? ? QuestionWhat happens when ionic bonds are formed? Which sentence contains a restrictive clause?We took a kayak to a beach where sea turtles hatch.The travel books, which I read in one day, were informative.Bird watchers often travel to Antarctica, which has so many penguins.The Dalai Lama, who travels extensively, recommends visiting new places. Bill Dollar is playing a video game. After level one he has - 17 points. You decide to challenge Bill online and after level one you have a score that is 29 points less than Bill's score. What is your score? 9. La crisis econmica que se desat en 1929, en Estados Unidos, tambin afect a los paseslatinoamericanos al reducirse sus exportaciones y, por consiguiente, sus importaciones. Para contrarrestar el efecto de esta crisis, los estados latinoamericanos adoptaron medidas como la sustitucin de importaciones, que consisti en:A. prohibir el ingreso de toda clase de productos extranjeros.B. otorgar total libertad de produccin, de cambio y de consumo.C. producir los bienes que tradicionalmente se traan del exterior.D. dejar actuar libremente las leyes de la oferta y la demanda. Un vehculo se mueve en lnea recta a una velocidad de 144 km/h durante 45 minutos. Qu distancia recorre? Dar la respuesta en metros.Ayudaaaaaa Sunsets are a deep red because A) tiny particles in the air are more efficient at scattering short wavelength light than they are at scattering long wavelength light. Hence, long wavelength light ends up coming directly towards you. B) most polluting gases and dust particles in the air are reddish in color and lend their color to that of the sky. C) air molecules absorb red light more efficiently than they do blue light because of their electron orbitals. D) air molecules absorb blue light more efficiently than they do red light because of their electron orbitals.